Last visit was: 28 Apr 2024, 12:47 It is currently 28 Apr 2024, 12:47

Close
GMAT Club Daily Prep
Thank you for using the timer - this advanced tool can estimate your performance and suggest more practice questions. We have subscribed you to Daily Prep Questions via email.

Customized
for You

we will pick new questions that match your level based on your Timer History

Track
Your Progress

every week, we’ll send you an estimated GMAT score based on your performance

Practice
Pays

we will pick new questions that match your level based on your Timer History
Not interested in getting valuable practice questions and articles delivered to your email? No problem, unsubscribe here.
Close
Request Expert Reply
Confirm Cancel
User avatar
Senior Manager
Senior Manager
Joined: 07 Jun 2004
Posts: 438
Own Kudos [?]: 2831 [1]
Given Kudos: 22
Location: PA
Send PM
User avatar
Director
Director
Joined: 18 Jul 2010
Status:Apply - Last Chance
Affiliations: IIT, Purdue, PhD, TauBetaPi
Posts: 538
Own Kudos [?]: 360 [0]
Given Kudos: 15
Concentration: $ Finance $
Schools:Wharton, Sloan, Chicago, Haas
 Q50  V37
GPA: 4.0
WE 1: 8 years in Oil&Gas
Send PM
User avatar
Senior Manager
Senior Manager
Joined: 24 Jun 2010
Status:Time to step up the tempo
Posts: 273
Own Kudos [?]: 673 [0]
Given Kudos: 50
Location: Milky way
Concentration: International Business, Marketing
Schools:ISB, Tepper - CMU, Chicago Booth, LSB
Send PM
avatar
Intern
Intern
Joined: 10 Apr 2008
Posts: 17
Own Kudos [?]: 9 [0]
Given Kudos: 4
Send PM
Re: A bag contains 3 white balls, 3 black balls & 2 red balls. [#permalink]
The possibilities are
WWR=2/56
WBR=3/56
WRR=1/56
BWR=2/56
BBR=3/56
BRR=1/56
RBR=1/56
RWR=1/56

Add all of them together you get 14/56=1/4.

Is there are better way to do this problem than working out all the combinations?
User avatar
Director
Director
Joined: 18 Jul 2010
Status:Apply - Last Chance
Affiliations: IIT, Purdue, PhD, TauBetaPi
Posts: 538
Own Kudos [?]: 360 [0]
Given Kudos: 15
Concentration: $ Finance $
Schools:Wharton, Sloan, Chicago, Haas
 Q50  V37
GPA: 4.0
WE 1: 8 years in Oil&Gas
Send PM
Re: A bag contains 3 white balls, 3 black balls & 2 red balls. [#permalink]
I see you are listing the combinations. If you listed all the combinations where in RRX (X=W/B) how many do we get? lets see
RRW = (2/8 x 1/7 x 3/6) = 1/56
RRB = 1/56
So (XYR) = 1-1/56-1/56 = 27/28
Goes back to my posting above.. Where is the mistake?
avatar
Intern
Intern
Joined: 10 Apr 2008
Posts: 17
Own Kudos [?]: 9 [0]
Given Kudos: 4
Send PM
Re: A bag contains 3 white balls, 3 black balls & 2 red balls. [#permalink]
You are saying (1 - possibility of RRX). But I think you also have to take into consideration the following combinations: RXX or XRX. Because these combinations won't give you a Red on the third pick, they should also be subtracted from 1.
User avatar
Director
Director
Joined: 18 Jul 2010
Status:Apply - Last Chance
Affiliations: IIT, Purdue, PhD, TauBetaPi
Posts: 538
Own Kudos [?]: 360 [0]
Given Kudos: 15
Concentration: $ Finance $
Schools:Wharton, Sloan, Chicago, Haas
 Q50  V37
GPA: 4.0
WE 1: 8 years in Oil&Gas
Send PM
Re: A bag contains 3 white balls, 3 black balls & 2 red balls. [#permalink]
Good point. So I should be looking at the combinations RRX RXX XRX that should cover all
So for P(RRX) = 2/8 x 1/7 x 6/6 = 2/56
P(RXX) = 3/8 x 6/7 x 5/6 = 15/56
P(XRX) = 6/8 x 2/7 x 5/6 = 10/56
I only got upto 2+15+10=27 so that is 1-27/56, I am missing 15 more cases so that 1-42/56 = 14/56=1/4
where did I go wrong now?
avatar
Intern
Intern
Joined: 14 May 2010
Posts: 16
Own Kudos [?]: 36 [0]
Given Kudos: 1
Schools:CBS
 Q51  V45
GPA: 4.0
Send PM
Re: A bag contains 3 white balls, 3 black balls & 2 red balls. [#permalink]
In terms of probability, the distribution at any time during the extraction process will reflect the initial distribution (this is because we are considering probability), so the answer is given by the initial distribution: 2/8=1/4.
avatar
Intern
Intern
Joined: 10 Apr 2008
Posts: 17
Own Kudos [?]: 9 [1]
Given Kudos: 4
Send PM
Re: A bag contains 3 white balls, 3 black balls & 2 red balls. [#permalink]
1
Kudos
Sorry. For got to mention P(XXX)

P(RRX) = 2/8 x 1/7 x 6/6 = 1/28
P(RXX) = 2/8 x 6/7 x 5/6 = 5/28
P(XRX) = 6/8 x 2/7 x 5/6 = 5/28
P(XXX) = 6/8 x 5/7 x 4/6 = 5/14

Add them together you get 21/28. 1-(21/28) = 1/4.
User avatar
Director
Director
Joined: 18 Jul 2010
Status:Apply - Last Chance
Affiliations: IIT, Purdue, PhD, TauBetaPi
Posts: 538
Own Kudos [?]: 360 [0]
Given Kudos: 15
Concentration: $ Finance $
Schools:Wharton, Sloan, Chicago, Haas
 Q50  V37
GPA: 4.0
WE 1: 8 years in Oil&Gas
Send PM
Re: A bag contains 3 white balls, 3 black balls & 2 red balls. [#permalink]
Excellent swdatta! P(XXX) how could I miss that! Going by what toshio is saying though, it seems the white and black are all misleading, all that matters is the initial probability.. so if that is really true, then we just calculate the initial value and are done!
avatar
Intern
Intern
Joined: 19 Nov 2009
Posts: 1
Own Kudos [?]: [0]
Given Kudos: 0
Send PM
Re: A bag contains 3 white balls, 3 black balls & 2 red balls. [#permalink]
the approach by toshio86 is better. I was going to post this approach but re-read the full thread and found it to be mentioned already.
My thought process was: since we dont dont know what was drawn out, the probability remains the same (even with or without replacement), unless any other skew was mentioned in the question.
Why this approach is better is because it does not depend on sequence or the number of ball drawn out (1st, 3rd or 8th etcetera). Imagine doing the selections for the fifth ball drawn and getting the same answer. Time saved = Better score.
Cheers
User avatar
Intern
Intern
Joined: 17 Apr 2010
Affiliations: ACCA
Posts: 26
Own Kudos [?]: 20 [0]
Given Kudos: 2
Concentration: Finance
Schools:IMD, Insead, LBS, IE, Cambridge, Oxford
GPA: 4.74 out of 5
Send PM
Re: A bag contains 3 white balls, 3 black balls & 2 red balls. [#permalink]
XXR=6/8*5/7*2/6
RXR=2/8*6/7*1/6
XRR=6/8*2/7*1/6
XXR+RXR+XRR=(6*5*2+2*6*1+6*2*1)/(8*7*6)=(5+1+1)/(4*7)=1/4 thus B
User avatar
Manager
Manager
Joined: 03 Aug 2010
Status:And the Prep starts again...
Posts: 84
Own Kudos [?]: 259 [0]
Given Kudos: 20
Concentration: IT Consulting
GMAT 2: 520
Send PM
Re: A bag contains 3 white balls, 3 black balls & 2 red balls. [#permalink]
I tried it using combinations and I got the answer wrong.

I used (6C2/8C2)*(2C1/6C1).

(6C2/8C2) - selection of first two balls
(2C1/6C1) - selection of one red ball from the remaining.

Please help.
Math Expert
Joined: 02 Sep 2009
Posts: 92977
Own Kudos [?]: 619696 [1]
Given Kudos: 81613
Send PM
Re: A bag contains 3 white balls, 3 black balls & 2 red balls. [#permalink]
1
Kudos
Expert Reply
ENAFEX wrote:
I tried it using combinations and I got the answer wrong.

I used (6C2/8C2)*(2C1/6C1).

(6C2/8C2) - selection of first two balls
(2C1/6C1) - selection of one red ball from the remaining.

Please help.


The simplest solution of this question is as follows: since the initial probability of drawing red ball is 2/8, then (without knowing the other results) the probability of drawing red ball will not change for ANY successive drawing: second, third, fourth... and will also equal to 2/8.

Answer: B.

Similar questions to practice:
a-box-contains-3-yellow-balls-and-5-black-balls-one-by-one-90272.html
each-of-four-different-locks-has-a-matching-key-the-keys-101553.html
a-box-contains-3-yellow-balls-and-5-black-balls-one-by-one-105990.html

OPEN DISCUSSION OF THIS QUESTION IS HERE: a-bag-contains-3-white-balls-3-black-balls-2-red-balls-100023.html

Archived Topic
Hi there,
This topic has been closed and archived due to inactivity or violation of community quality standards. No more replies are possible here.
Where to now? Join ongoing discussions on thousands of quality questions in our Problem Solving (PS) Forum
Still interested in this question? Check out the "Best Topics" block above for a better discussion on this exact question, as well as several more related questions.
Thank you for understanding, and happy exploring!
GMAT Club Bot
Re: A bag contains 3 white balls, 3 black balls & 2 red balls. [#permalink]
Moderators:
Math Expert
92977 posts
Senior Moderator - Masters Forum
3137 posts

Powered by phpBB © phpBB Group | Emoji artwork provided by EmojiOne